Q19-OG 13th CR

This topic has expert replies
Newbie | Next Rank: 10 Posts
Posts: 4
Joined: Tue May 29, 2012 7:56 am

Q19-OG 13th CR

by rupalikunmun123 » Fri Aug 17, 2012 11:08 am
A major network news organization experienced a drop in viewership in the week following the airing of a controversial report on the economy. The network also received a very large number of complaints regarding the report. The network, however, maintains that negative reactions to the report had nothing to do with its loss of viewers.

Which of the following, if true, most strongly supports the network's position?
A.The other major network news organizations reported similar reductions in viewership during the same week.
B.The viewers who registered complaints with the network were regular viewers of the news organization's programs.
C.Major network news organizations publicly attribute drops in viewership to their own reports only when they receive complaints about those reports.
D.This was not the first time that this network news organization has aired a controversial report on the economy that has inspired viewers to complain to the network.
E. Most network news viewers rely on network news broadcasts as their primary source of information regarding the economy.


Can anyone please explain why E is wrong? I assumed that E is correct because most network news viewers rely on the network news for primary source of information so the viewership will not drop as the viewers will have no more alternatives to get the information regarding the economy.

Please correct me if my understanding is wrong.

User avatar
GMAT Instructor
Posts: 1052
Joined: Fri May 21, 2010 1:30 am
Thanked: 335 times
Followed by:98 members

by Patrick_GMATFix » Fri Aug 17, 2012 4:59 pm
Hi rupalikunmun123,

The problem with the justification you gave for E is that you assumed that there is only one network news station. Even if most viewers rely on network broadcasts, they could have abandoned this network as a result of the negative report and switched to another network. Thus the information in E doesn't do a good job of showing that the drop of viewership had little to do with the negative report.

-Patrick
  • Ask me about tutoring.

User avatar
Senior | Next Rank: 100 Posts
Posts: 56
Joined: Mon Feb 07, 2011 7:57 am
Thanked: 1 times
GMAT Score:600

by crisro » Sat Aug 18, 2012 6:32 pm
IMO A is the only possible correct answer.

User avatar
Master | Next Rank: 500 Posts
Posts: 130
Joined: Fri Apr 20, 2012 8:13 am
Location: Toronto, Ontario
Thanked: 16 times
Followed by:4 members
GMAT Score:650

by tisrar02 » Mon Aug 20, 2012 10:09 am
IMO: A

Evidence: Controversial report was aired and the number of viewers dropped

Conclusion: The report is not in direct correlation to the drop in viewers

Assumption: There is something else that must have led to the drop in viewership.

A- Sounds somewhat decent- Must be something else that declined viewership
B- Weakens the argument- Regular viewers were the ones to leave so the report must have had an effect.
C- Weakens the argument- Reports like these effect the viewership all the time so this weakens the networks statement
D- Irrelevant- Only speaks about complaining about the report.
E- Does not strengthen the fact that viewership has a direct correlation to the new report.
Dedication is what leads to success...

User avatar
Senior | Next Rank: 100 Posts
Posts: 83
Joined: Sun Aug 19, 2012 12:42 am

by hjafferi » Thu Aug 23, 2012 5:20 am
IMO a

Only reason which justifies that drop in viewership was not because of the report.

Legendary Member
Posts: 774
Joined: Mon Jan 23, 2012 4:32 am
Thanked: 46 times
Followed by:14 members

by aditya8062 » Thu Feb 21, 2013 10:48 pm
i have a following question regarding option A : how can we assume in option A that "the other major network news organization" did not air the controversial report ? what if the same report was also aired by "other major networks"(i mean its not something unusual .it does happen that two news channel air the same report ) and that "the report " was a real reason behind the reduction of their viewers .i feel that option A is incomplete in its text .i do agree that in causal questions we have to show the specifics of cause and effect but here in option A we are just referring to the effect without even discussing the presence or absence of cause!!
i guess option A is making unwarranted assumptions
experts please help
PS :: the OA : is A .i am not questioning the OA .but whats bothering me is that option A is making the assumption that i have stated above and i have read in many post of Ron that the answers in strengthening and weakening questions sud stand of their own and sud not make any kind of assumption
instructors plz help me understand
thanks and regards

GMAT/MBA Expert

User avatar
GMAT Instructor
Posts: 451
Joined: Thu Jan 21, 2010 11:58 am
Location: New York City
Thanked: 188 times
Followed by:120 members
GMAT Score:770

by Tommy Wallach » Fri Feb 22, 2013 2:45 pm
Hey Aditya,

Honestly, it isn't possible/logical for multiple news stations to air the same report. It would be a bit like having a TV network air a new episode of a TV show. Other channels simply can't post the exact same thing; it isn't how TV works. Does this require a bit of a working knowledge? Yes, I suppose it does. But it's inferable enough (and don't forget, no other answer choice is better!).

-t
Tommy Wallach, Company Expert
ManhattanGMAT

If you found this posting mega-helpful, feel free to thank and/or follow me!

Legendary Member
Posts: 1404
Joined: Tue May 20, 2008 6:55 pm
Thanked: 18 times
Followed by:2 members

by tanviet » Sat Feb 23, 2013 1:24 am
aditya8062 wrote:i have a following question regarding option A : how can we assume in option A that "the other major network news organization" did not air the controversial report ? what if the same report was also aired by "other major networks"(i mean its not something unusual .it does happen that two news channel air the same report ) and that "the report " was a real reason behind the reduction of their viewers .i feel that option A is incomplete in its text .i do agree that in causal questions we have to show the specifics of cause and effect but here in option A we are just referring to the effect without even discussing the presence or absence of cause!!
i guess option A is making unwarranted assumptions
experts please help
PS :: the OA : is A .i am not questioning the OA .but whats bothering me is that option A is making the assumption that i have stated above and i have read in many post of Ron that the answers in strengthening and weakening questions sud stand of their own and sud not make any kind of assumption
instructors plz help me understand
thanks and regards
I also choose a and I think exactly like you.

in many cases, strengthener/weakener require an assumption as in this case. I call this situation, OA NEED ASSUMPTION.

e gmat expert also talk about this problem in a posting in gmatclub. I think you can find it on the internet.

this situation is what we have to accept/learn for gmat.

Legendary Member
Posts: 774
Joined: Mon Jan 23, 2012 4:32 am
Thanked: 46 times
Followed by:14 members

by aditya8062 » Sat Feb 23, 2013 1:35 am
to duongthang
kindly attach the link that u talking of
thanks and regards
aditya

Legendary Member
Posts: 1404
Joined: Tue May 20, 2008 6:55 pm
Thanked: 18 times
Followed by:2 members

by tanviet » Sat Feb 23, 2013 2:47 am
According to the Tristate Transportation Authority

pls seach for above text in gmatclub, around which the relevan discusion will be found

GMAT/MBA Expert

User avatar
GMAT Instructor
Posts: 451
Joined: Thu Jan 21, 2010 11:58 am
Location: New York City
Thanked: 188 times
Followed by:120 members
GMAT Score:770

by Tommy Wallach » Sun Feb 24, 2013 2:46 pm
I would just remind everyone that the rule is you're picking the "best" answer, not the "perfect" answer. So don't stress so much about whether the answer requires an assumption or not, but if there's an answer choice that doesn't require an assumption. If there isn't, then you're still picking the best answer.

-t
Tommy Wallach, Company Expert
ManhattanGMAT

If you found this posting mega-helpful, feel free to thank and/or follow me!